LSAT and Law School Admissions Forum

Get expert LSAT preparation and law school admissions advice from PowerScore Test Preparation.

 Administrator
PowerScore Staff
  • PowerScore Staff
  • Posts: 8919
  • Joined: Feb 02, 2011
|
#91417
Complete Question Explanation

The correct answer choice is (D).

Answer choice (A):

Answer choice (B):

Answer choice (C):

Answer choice (D): This is the correct answer choice.

Answer choice (E):

This explanation is still in progress. Please post any questions below!
User avatar
 ashpine17
  • Posts: 321
  • Joined: Apr 06, 2021
|
#92016
So this question took me a bit. I had to draw out multiple diagrams.

I put V in second place on my chart and then I used the S-O-R chain to draw out several boards. If S is in 3, then it becomes PVSOR and if S is in 1, then it becomes SVPOR or SVORP. FOr the first board, it was FIIFI and for the second it's F/I F/I IFI and for the last, it was F/I IFIF and I can eliminate and get answer choice D.
 Rachael Wilkenfeld
PowerScore Staff
  • PowerScore Staff
  • Posts: 1358
  • Joined: Dec 15, 2011
|
#92402
Hi ashpine,


My three diagrams were order PVSOR and, SVORP then SVPOR. SVOPR wouldn't work because both O and P would be fertalized in that situation, and they can't be consecutive. I drew really rough sketches, but I wanted to make sure that I covered all the possibilities.

So with the three diagrams I drew, I can analyze the different answer choices.

Answer choice (A) would be false in the second (SVORP) diagram.
Answer choice (B) would be false in the first (PVSOR) diagram.
Answer choice (C) could be false in the thirt (SVPOR) diagram.
Answer choice (D) is true in all three diagrams.
Answer chioce (E) could ber false in the third (SVPOR) diagram.

It can take a few seconds to draw out the diagrams, but it makes those answer choices so much faster. The less you have to keep in your head, the faster you can move. So while I totally understand that it's a little slow and plodding in the moment, it's worth the payoff in both speed and accuracy in evaluating answers.
 jacquelinekier
  • Posts: 2
  • Joined: May 24, 2022
|
#96653
Hi,

I am a little confused on how D is correct. I made two templates; SVORP and PVSOR. The latter allows for D to be correct, given that O would be Fertilized in spot 4. However, in the template of SVORP, V is before R and thus O would be fertilized in spot 3. Since P is in spot 5 (not spot 3) it must be irrigated. R in spot 4 cannot be Fertilized because O is fertilized in spot 3. So neither of the last two crops would be fertilized. Let me know what I ma getting wrong here, thank you!
 jacquelinekier
  • Posts: 2
  • Joined: May 24, 2022
|
#96654
jacquelinekier wrote: Tue Aug 09, 2022 10:33 pm Hi,
I realized my mistake, I misread rule 3.

Get the most out of your LSAT Prep Plus subscription.

Analyze and track your performance with our Testing and Analytics Package.